PAST PAPER QUESTIONS FOR MAY EXAMS - Elgin · PDF filePAST PAPER QUESTIONS FOR MAY EXAMS Of...

24
STANDARD GRADE INTERMEDIATE HIGHER ADV HIGHER ENGLISH EXAM SUPPLEMENT FEATURING PAST PAPER QUESTIONS FOR MAY EXAMS

Transcript of PAST PAPER QUESTIONS FOR MAY EXAMS - Elgin · PDF filePAST PAPER QUESTIONS FOR MAY EXAMS Of...

Page 1: PAST PAPER QUESTIONS FOR MAY EXAMS - Elgin · PDF filePAST PAPER QUESTIONS FOR MAY EXAMS Of cial SQA past paper questions for Maths, Biology, Physics and Chemistry - from Standard

STA

ND

AR

D G

RA

DE

IN

TER

MED

IATE

HIG

HER

AD

V H

IGH

ER

ENGLISH

EXAM SUPPLEMENT FEATURING

PAST PAPER QUESTIONS FOR MAY EXAMS

Page 2: PAST PAPER QUESTIONS FOR MAY EXAMS - Elgin · PDF filePAST PAPER QUESTIONS FOR MAY EXAMS Of cial SQA past paper questions for Maths, Biology, Physics and Chemistry - from Standard

2 | Official SQA Past Papers Supplement - English 2 | Official SQA Past Papers Supplement - Mathematics

FREE EXAM GUIDESPAST PAPER QUESTIONS FOR MAY EXAMS

Official SQA past paper questions for Maths, Biology, Physics and Chemistry

- from Standard to Advanced Higher grades

Tomorrow 18 March Monday 19 March Tuesday 20 March

DGM Training Consultants offer a first class recruitment and training service.

They offer modern apprenticeship schemes fully funded by Skills Development Scotland.

If you are aged between 16 and 24 and are interested in a career in admin, customer services or financial services

then make the connection through DGM.

With over 20 years in the training industry, DGM prides itself as being one of Edinburgh's

most experienced training providers.

DGM Training 9-11, Maritime StreetEdinburgh EH6 6SBTel: 0131 625 1021

"Thinking about your optionsonce you have left school?"

dgmtraining consultants

Page 3: PAST PAPER QUESTIONS FOR MAY EXAMS - Elgin · PDF filePAST PAPER QUESTIONS FOR MAY EXAMS Of cial SQA past paper questions for Maths, Biology, Physics and Chemistry - from Standard

Official SQA Past Papers Supplement - English | 3

Use SCOT2 at www.brightredpublishing.co.uk for 30% off and free post and packaging!

STANDARD GRADE I INTERMEDIATE I HIGHER I ADV HIGHER

ENGLISH

EXAM SUPPLEMENT FEATURING

FREE IN TOMORROW’S

English is the study of verbal and written

communication in Scotland’s main language.

It teaches you how to get the most out of reading,

writing, listening and talking and through your

studies you will become familiar with the richness of

literature as well as gaining vital skills. These skills

will equip you to study and work in a world in which

English has become the dominant language of

international communication.

Any English course,

whatever its stage or level,

seeks to enrich the lives

of students, contribute to

their personal and social

growth and enable them

to develop to the highest

possible level the skills

associated with listening

and talking, reading and

writing. In so doing, it

should make you aware

of the main ways in which language works in your life. Within this context,

you will be made aware of Scotland’s cultural diversity and

the contribution of minority cultures. You will be encouraged to

develop qualities such as resourcefulness, cooperativeness,

ambition and mutual respect.

In the provision of these contexts in an English course, literary texts are of

central importance. They offer the vicarious experiences through which your

teacher will pursue the main, subtly-linked aims of English.

If you are to achieve the linked aims of linguistic development and personal

enrichment described above, it is important that you should have some

experience of the work of Scottish writers.

No matter which level you are studying (Advanced Higher, Higher,

Intermediates or Standard Grade), your performance in the SQA exam will

determine your overall award. This means that you need to practise the skills

that you will need. Effective use of Past Papers is the best way to do this as

this will allow you to become familiar with the sort of questions asked in the

exam. Don’t forget to practise some questions “against the clock” so that you

get used to dealing with the time pressure of the exam. You should create your

own study plan as you prepare for the exam. What this looks like will depend

on your course but will probably include the following:

Advanced Higher Revision of Literary Study textsPractising essays (1 hour 30 minutes maximum)Practising Textual Analysis or other “option” questions (1 hour 30 minutes maximum)

Higher/IntermediateRevision of Close Reading conceptsPractising Close Reading (1 hour 45 minutes maximum) Revision of prose, poetry, drama and media textsPractising critical essays (45 minutes maximum for each one)

Standard GradeRevision of types of Reading questionsRevision of techniques (imagery, sound, sentence structure)Practice with Past Papers (50 minutes max. for Reading)Timed writing (1 hour 15 minutes max.)

Don’t leave your revision to the last minute – allocate a set amount of time per night or per week for study/revision and stick to it. Don’t forget to use any “study periods” you have in school effectively. Don’t sit for hours just staring at books – write down notes or quotations; make notes in margins; underline or highlight sections of text. Write things you need to remember on “post-its” and stick them round the house. Use your phone as another way of storing materials – but remember that you must not take it into the exam! Ask your class teacher if you are unsure about anything and ask again if you still “don’t get it”.

Project manager Tim DonaldDesign Ross Burgess

WITH ANSWERS

OFFICIALPAST

PAPERSSQA

Past Paper questions © Scottish Qualifications Authority. Text and past paper answers © Bright Red Publishing

Page 4: PAST PAPER QUESTIONS FOR MAY EXAMS - Elgin · PDF filePAST PAPER QUESTIONS FOR MAY EXAMS Of cial SQA past paper questions for Maths, Biology, Physics and Chemistry - from Standard

4 | Official SQA Past Papers Supplement - English

STANDARD GRADE

ENGLISHAbout the author

Iain Valentine is principal teacher of English at Elgin Academy. He has taught English at all levels for 24 years and was the SQA’s Principal Assessor for Standard Grade English from 2007 – 2011.

The Standard Grade qualification in English develops your ability to read, write, listen and talk effectively. Having completed this course, you will have gained an understanding of how the language works, from sentence construction to the communication of ideas through imaginative writing. You will also have been introduced to the variations of the English language. You will have gained key skills for functioning in society: understanding information and being able to communicate in verbal and written forms.

The Standard Grade English Course is at three levels:

Standard Grade FoundationStandard Grade GeneralStandard Grade Credit

Exam timetable - English Course Date Time

Standard Grade – F/G/C Writing Tuesday 26th April 9am – 10.15am Standard Grade – Foundation Reading Tuesday 26th April 10.35am – 11.25am Standard Grade – General Reading Tuesday 26th Apri 1pm – 1.50pm Standard Grade – Credit Reading Tuesday 26th April 2.30pm – 3.20pm

Intermediate 1 – Close Reading Wednesday 16th May 9am – 10.00am Intermediate 1 – Critical Essay Wednesday 16th May 10.20am – 11.05am Intermediate 2 – Close Reading Wednesday 16th May 1pm – 2pm Intermediate 2 – Critical Essay Wednesday 16th May 2.20pm – 3.50pm

Higher – Close Reading Thursday 17th May 9am – 10.45am Higher – Critical Essay Thursday 17th May 11.05am – 12.35pm

Advanced Higher Thursday 17th May 1pm – 4pm

Page 5: PAST PAPER QUESTIONS FOR MAY EXAMS - Elgin · PDF filePAST PAPER QUESTIONS FOR MAY EXAMS Of cial SQA past paper questions for Maths, Biology, Physics and Chemistry - from Standard

Official SQA Past Papers Supplement - English | 5

Use SCOT2 at www.brightredpublishing.co.uk for 30% off and free post and packaging!

The Folio

Your five best pieces of work (two pieces of Writing and three pieces of work based on Reading, covering at least two genres) will have been sent to SQA for marking by the end of March. The grades your folio pieces are given will make up half of your overall grades for Reading and Writing.

Talking

Remember that your class teacher is responsible for assessing your per-formance in Talking. By now, your teacher will have sent your grade for Talking to the SQA.

The Reading Exam

You will sit two Reading exams: the General and Credit papers or the General and Foundation papers. Each exam paper consists of a passage and a series of questions about the passage. The passage can be fiction (an extract from a novel or a short story) or non-fiction (an article from a newspaper or magazine; an extract from an autobiography; travel writing or any other kind of longer non-fiction text).

There are usually between 19-25 questions to answer and the total number of marks is always 50. To have a good chance of achieving the upper grade in each paper, you will need to achieve at least 70% of the available marks (35/50); to achieve the lower grade, you might need 50% (25/50). The exam only lasts for 50 minutes so you will need to get used to reading passages quickly and accurately!

Some general advice for the Reading papers

. read the whole passage before you start writing your answers in the booklet provided (ignore the people beside you who start scribbling straight away)

. underline or highlight parts of the passage to help with answering in your own words (see below)

. follow the instructions before each group of questions about where to look for an answer (e.g.“Look at Paragraphs 6-8”)

. only write the required answer – you don’t need to answer in sentences

. if a question asks you to Quote (Credit) or Write down an expression (General and Foundation), you must find the appropriate words in the passage and write them in your answer booklet

. when a question asks you to “Explain fully …” you must provide as detailed a response as you can (remember that all questions are worth 2 marks)

. if you wish to change an answer, score out your first answer and then write the new answer above or below

. you must answer all the questions - if you are still stuck on anything just before time is up, make your best guess

Type of QuestionOwn words

Example from Past PaperStephen Woollard, the zoo’s education manager, is both “proud” and “delighted” aboutthe new enclosure. (Paragraph 4)In your own words, explain why he is both “proud” and “delighted”.

How to tackle this kind of questionUse the locate and translate method. Find the information in the passage, underline or highlight it and then turn

it into your own words. So the words “he helped design” in the passage become something like “he was involved

in planning” and the words“realised so spectacularly” become something like “it worked out so well” or “the

dream came true” in your answer. If you don’t use your own words in your answer you will score 0.

2010 - General, Question 8 Paragraph 4

‘“They’ve moved from an ordinary house to a million-aire’s mansion,” beams Stephen Woollard, as he shows me around the place, justifiably proud of the structure he helped design. The education manager from the Royal Zoological Society of Scotland says the idea of a network that allowed scientists to study chimps in something like their natural environment was first proposed in the 1960’s. He seems delighted this has finally been realised so spectacularly.’

Advice on particular types of questions is below

Page 6: PAST PAPER QUESTIONS FOR MAY EXAMS - Elgin · PDF filePAST PAPER QUESTIONS FOR MAY EXAMS Of cial SQA past paper questions for Maths, Biology, Physics and Chemistry - from Standard

6 | Official SQA Past Papers Supplement - English

Type of QuestionTechniques (sentence structure)

Type of QuestionLink

Example from Past Paper2010 - Credit, Question 7Explain the use of dashes in “ ... –rat or mouse– ...” (Paragraph 3)

Example from Past Paper“Therein lies the problem, though.” (Paragraph 8)Explain how this sentence acts as a link between Paragraphs 7 and 8.

How to tackle this kind of questionYou should be able recognize when a writer is using a long sentencea short sentencea minor sentence a listrepetitioninversionclimaxparenthesis (usually indicated by paired dashes or brackets)

These questions are often easier than they seem with a mark often given simply for identifying what you see. For this example you would score 1 mark for explaining that the dashes indicate a parenthesis or that they add more informa-tion . The second mark would be awarded for saying that the parenthesis tells us about the types of creatures which might have been there.

How to tackle this kind of questionFind and quote the words in the sentence which refer back to what the writer has said in the previous paragraph

(Therein) and then find and quote the words which introduce the next point the writer makes (the problem). You

should also look out for words like yet, but, however … when answering this kind of question.

2010 - Credit, Question 7

‘He moved quietly about the field, amazed at the silence. No whisper of wind, no rustle of creature-rat or mouse-moving about.’

2011 – Credit, Question 9 Paragraphs 7 and 8

‘Back in Glasgow, it’s only the grey tower blocks on the skyline and the cranes of the Clyde shipyards that remind you this isn’t a Paris backstreet or downtown New York. Parkour, say its practitioners, transgresses physical, men-tal, cultural and geographical boundaries. It is unique, operates off the radar and involves risk and a sense of dan-ger. Just as city kids of the late 1970s and early 1980s found creativity in skate parks and hip-hop, it isn’t difficult to see why, for some, parkour is now synonymous with freedom and cool.

Therein lies the problem, though. The glamourisation of parkour has been a catalyst for its growth but has also com-municated mixed messages. The explosion in popularity has caused a schism to develop within the parkour com-munity over the movement’s philosophy. Is it, for example, about dangerous jumps across tenements, and the sort of flips and tricks which have seen brand-name executives reaching for their cheque books? Or is it, as many argue, about fine-tuning the mind and body to overcome obstacles and fear?’

Type of QuestionTechniques (imagery and sound)

Example from Past Paper2010 - Credit, Question 6“The moon made a white road across the distant sea.” (Paragraph 2)(a) What technique is used in this expression? (b) Explain fully what this expression suggests about the moonlight.

How to tackle this kind of questionYou need to be able to identify at least the following techniques:similemetaphoralliterationonomatopoeiapersonification

For this example, to answer part (a) you need to be able to identify that the writer is using a metaphor (“made a white road”). To answer part (b) you have to think about what this image suggests. In your answer you could say it suggests that it is straight or long or that it stretches into the distance. You could also say that the image sug-gests the moonlight has a “solid” quality or is very bright or provides a contrast to the surrounding darkness. The question asks you to “explain fully” so you have to give at least two pieces of information in your answer. Don’t forget that at Credit level you will usually be asked to say something about the effectiveness or appropriateness of the technique as well as identifying it.

2010 - Credit, Question 6

‘The moonlight shimmered across the stooks so that they looked like men, or women who had fallen asleep upright. The silence gathered around him, except that now and again he could hear the bark of a dog and the noise of the sea. He touched the stubble with his finger and felt it sharp and thorny as if it might draw blood. From where he was he could see the lights of the houses but there was no human shape to be seen anywhere. The moon made a white road across a distant sea.’

Page 7: PAST PAPER QUESTIONS FOR MAY EXAMS - Elgin · PDF filePAST PAPER QUESTIONS FOR MAY EXAMS Of cial SQA past paper questions for Maths, Biology, Physics and Chemistry - from Standard

Official SQA Past Papers Supplement - English | 7

Use SCOT2 at www.brightredpublishing.co.uk for 30% off and free post and packaging!

Type of QuestionMeaning plus context

Example from Past Paper“The main problem for us is bureaucracy.” (Paragraph 15)Show how the context helps you understand the meaning of “bureaucracy”.

How to tackle this kind of questionIf you don’t know the meaning of the word already, look at the words around it (the context) and see if they offer any

clues. In your answer you must give the meaning of the word and then quote the other words which helped you to

arrive at that meaning. So you would write that “bureaucracy” means official procedures or paperwork and the con-

text which helped you understand this were the expressions “liability insurance” or “disclaimers” or “risk assessment”.

2011 – Credit, Question 19 Paragraph 15

‘In the 18 months since he founded Glasgow Parkour Coaching, assisted by fellow coaches Mick McKeen, Gavin Watson and David Lang, Grant says he has seen only one injury. “It happened over there,” he says, pointing to a row of innocuous wooden posts. In front of him, traceurs andtraceuses from tonight’s class are poised like trapeze art-ists on railings completing a study in balance. “The main problem for us is bureaucracy and the persistent idea that this is a dangerous activity,” says Grant. “People think parkour is just about jumping off walls and they have trouble seeing the outcomes and rewards. We have liability insurance—we do risk assessments and we get people to sign disclaimers. We are serious about what we do. I don’t sleep sometimes because, as a coach, I am responsible for other people.”’

Type of QuestionWord choice

Example from Past PaperComment on the writer’s use of word choice to show the agility of the “athletic youths”.

How to tackle this kind of questionThe first thing you must do in any word choice question is to quote the words used by the writer to create a par-

ticular effect.

In this example there were many different words and expressions you could choose. You would write down

“bound” or “sure-footed” or “leaping” or “like cats” or “trainers crunching into the gravel” or “roll on to their

shoulders” or “springing” or “pushing off” or “fluid” or “unbroken” or “up and over a wall” or “vaulting”.

Then you have to add a suitable comment. In this case you could say that any one of your chosen words or expres-

sions suggests the speed or energy or flexibility of the youths.

2011 - Credit, Question 2

‘It is Wednesday night in Glasgow. The high walls, rails and steps of Rottenrow Gardens look like some form of municipal amphitheatre under the reddening sky. Several athletic youths in T-shirts and jogging bottoms are moving quickly. They bound over rocks, sure-footed, before leap-ing like cats into the air, their trainers crunching into the gravel on landing. To move off again, they roll on to their shoulders on the hard ground, springing up and pushing off in one fluid, unbroken movement. You can still see dust in the air as they pass on through the shadows, up and over a wall or vaulting a railing.

Witnessing this for the first time, you might think you’ve come across an unorthodox piece of urban theatre, and in a sense you have. This is parkour, an underground activity that started in the suburbs of Paris in the 1980s and is now sweeping Europe, fuelled by the Internet, especially DIY productions on video sharing websites.’

Type of QuestionContrast

Example from Past PaperIn Paragraph 5, the writer develops a contrast between Briony and her big sister.(a) In your own words, state what the contrast is.

How to tackle this kind of questionWhen a question asks you about contrast you must make sure you always refer to the two sides of the contrast

and say what is being contrasted with what. In this case you had to identify that Briony seems to be organised or

obsessive while her sister seems messy or careless.

2008 - Credit, Question 10 (a)

‘She was one of those children possessed by a desire to have the world just so. Whereas her big sister’s room was a stew of unclosed books, unfolded clothes, unmade bed, unemptied ashtrays, Briony’s was a shrine to her controlling demon: the model farm spread across a deep window ledge consisted of the usual animals, but all facing one way-towards their owner-as if about to break into song, and even the farmyard hens were neatly corralled.

In fact, Briony’s was the only tidy upstairs room in the house. Her straight-backed dolls in their many-roomed mansion appeared to be under strict instructions not to touch the walls; the various thumb-sized figures to be found standing about her dressing table-cowboys, deep-sea divers, humanoid mice-suggested even by their ranks and spacing a citizen army awaiting orders.’

Page 8: PAST PAPER QUESTIONS FOR MAY EXAMS - Elgin · PDF filePAST PAPER QUESTIONS FOR MAY EXAMS Of cial SQA past paper questions for Maths, Biology, Physics and Chemistry - from Standard

8 | Official SQA Past Papers Supplement - English

Writing

The Writing exam lasts for one hour and 15 minutes. In the exam paper there are usually between 19 and 23 assignments and some pictures to help you think about what you have to write. You have to attempt only ONE assignment.

Because you only have 75 minutes in the exam, you should allocate your time as follows:

. five minutes to read through the booklet, look at the pictures and make your choice of assignment. a further ten minutes to think about and PLAN your piece of writing. one hour to produce your piece of writing (including a quick look over your work before time is up)

There will be a wide variety of assignments to choose from. There will be tasks which require you to

. convey information . deploy ideas (these questions will usually include the words “Give your views”). describe personal experience, feelings, reactions (these questions will usually include the words “Remember to include your thoughts and feelings”). write a story (these questions also contain the instruction to “develop setting and character as well as plot”). write in any way you choose using a picture as a stimulus for your ideas (for these assignments you can choose the kind of writing you want to do ). describe the scene suggested by a quotation from poetry or a short piece of prose.

Once you have decided on which assignment you are attempting, PLAN your work carefully. Use whatever kind of plan works best for you. This might be a “mind-map” or “spider-diagram” or it might just be a list of ideas. Only once you’ve got these details clear in your own mind should you begin – again you mustn’t be put off by the scribblers sitting beside you who start writing straight away.

Let’s look at some specific advice about how you might have approached some of the most popular types of assignments in recent SQA exam papers.

Type of QuestionContinuing an idea

Example from Past Paper“Another was a passion for secrets.” (Paragraph 6)By referring to the passage, show how the writer continues this idea in the rest of the paragraph.

How to tackle this kind of questionLook for words or expressions which suggest a similar idea and which follow the words referred to in the question.

In this case you would gain marks for writing down “secret drawer” or “locked diary” or “code” or “safe” or “secret

numbers” – all of which continue the idea of the character’s liking for secrets.

2008 - Credit, Question 12

‘A taste for the miniature was one aspect of an orderly spirit.

Another was a passion for secrets: in a prized varnished

cabinet, a secret drawer was opened by pushing against the

grain of a cleverly turned dovetail joint, and here she kept a

diary locked by a clasp, and a notebook written in a code of

her own invention. In a toy safe opened by six secret numbers

she stored letters and postcards. An old tin petty cash box

was hidden under a removable floorboard beneath her bed.

In the box were treasures that dated back four years, to her

ninth birthday when she began collecting: a mutant double

acorn, fool’s gold, a rain-making spell bought at a funfair, a

squirrel’s skull as light as a leaf.’

Deploy ideas

Type of Assignment Example from Past Paper

2011 – Assignment 12Reading books on a screen will never replace the goodold-fashioned paperback.Give your views.

How to tackle this type of assignment

In the limited time available to you in the exam, it’s best to keep this kind of essay relatively simple. An obvious structure would be . introduction. advantages of “traditional” books (no need for power source; easy to flick through; look good on a shelf; you can read them in the bath without worrying about costly damage; something you can touch, feel, smell …). advantages of e-books (convenient; easy to store; can be downloaded very quickly; cost; ideal for going on holiday … ). conclusionYou should make use of effective linking words and expressions to join the various parts of your essay together (however, in addition to this, despite this, in conclusion…). Try to use the kind of sentence structure techniques you have learned about in your work for the Reading paper (list, repetition, rhetorical question…)

Page 9: PAST PAPER QUESTIONS FOR MAY EXAMS - Elgin · PDF filePAST PAPER QUESTIONS FOR MAY EXAMS Of cial SQA past paper questions for Maths, Biology, Physics and Chemistry - from Standard

Official SQA Past Papers Supplement - English | 9

Use SCOT2 at www.brightredpublishing.co.uk for 30% off and free post and packaging!

Some general advice

. If you choose a “Give your views” type question, make sure it’s a subject you know and have opinions about

. if you choose a “Write in any way you choose” type question, your writing must have something to do with the picture

. Don’t mix up writing a story with writing about a personal experience

. Don’t learn an essay off by heart and try to reproduce it in the exam

. Pay attention to spelling and punctuation (especially punctuation of direct speech – ask your teacher about this if you need a refresher)

. Remember that you are writing “under pressure” in the exam – your writing doesn’t have to be “perfect” but sentence construction

must be accurate and formal errors will not be “significant” if you are to achieve a credit grade

. Be consistent with the tenses of verbs (keep things in the past or the present but don’t mix them up)

. Organise your writing into paragraphs

. Make sure your handwriting is neat and legible – you don’t want to make your marker’s job more difficult than it needs to be

Describe personal experience

2011 – Assignment 10Write about an occasion when science or technology changedyour life.Remember to include your thoughts and feelings.

As the wording of the assignment suggests, you should not just say what happens to you. You must write about how the experience affected you emotionally. Were you unhappy? Scared? Relieved? Excited? Think about what you have learned from this experience. Try to make use of techniques such as effective similes and metaphors as part of your descrip-tion.Think about interesting ways to structure writing like this. For example you might start at the end of the experience and then have a “flashback” to how it all started.

Type of Assignment Example from Past Paper How to tackle this type of assignment

Write a story 2011 – Assignment 9Write a short story using one of the following titles:

The Experiment

The Monster

You should develop setting and character as well as plot.

You will need to think about where and when your story is set; who the characters are; how it begins, develops and ends. The best stories will limit the setting to one or two places; have only a few characters; include some kind of turning point or climax and also end in a memorable or surprising fashion. The best writing in the exam will be “clear and stylish” so make sure you are “manipulating language to achieve particular effects” – think about how you might include techniques such as simile, metaphor, personification, alliteration and onomatopoeia.

Bright Red Results Standard Grade EnglishAuthor: David CockburnISBN: 978-1-906736-09-5

£8.99

Bright Red Official Past Papers Standard Grade Foundation/General English

ISBN: 978-1-84948-166-3£8.99

Bright Red Official Past Papers Standard Grade General/Credit English

ISBN: 978-1-84948-167-0£8.99

Page 10: PAST PAPER QUESTIONS FOR MAY EXAMS - Elgin · PDF filePAST PAPER QUESTIONS FOR MAY EXAMS Of cial SQA past paper questions for Maths, Biology, Physics and Chemistry - from Standard

10 | Official SQA Past Papers Supplement - English

INTERMEDIATE 1 AND 2

ENGLISHAbout the author

David Swinney is principal teacher of English at Knox Academy, Haddington and is also the SQA’s Principal Assessor for Intermediate 1 and Intermediate 2 English.

The Intermediate 1 and Intermediate 2 qualifications in English help you to develop your communication and understanding of the written and spoken word. Each is a valuable qualification in its own right, but Intermediate 1 is a good preparation for Intermediate 2, and Intermediate 2 leads on well to Higher English.

In your May exam there will be two papers. The first one, Close Reading, is all about testing the skills you have learned in Reading. It is a skills-based exam - to prepare for it you need to practise these skills. The second paper, Critical Essay, assesses your reading of the texts you have studied and enjoyed throughout the year (literature, media or language). It also tests your ability to write an essay. For Intermediate 1 you will have to write one essay, for Intermediate 2 you will have to write two (on two different texts). This second paper is the ‘swotting up’ one. You have to go into the exam knowing your texts well and feeling confident enough to refer to them in some detail (yes, including quotes!).

Page 11: PAST PAPER QUESTIONS FOR MAY EXAMS - Elgin · PDF filePAST PAPER QUESTIONS FOR MAY EXAMS Of cial SQA past paper questions for Maths, Biology, Physics and Chemistry - from Standard

Official SQA Past Papers Supplement - English | 11

Use SCOT2 at www.brightredpublishing.co.uk for 30% off and free post and packaging!

THE CLOSE READING EXAM

At Intermediate 1 and 2, the Close Reading exam paper lasts for one hour and the exam

paper will have one passage. The passage will be selected from a work of non-fiction or

from quality journalism and the total length will be in the region of 1,000 words. There is

usually a brief introduction (printed in italics) just before the passage begins. This can be

very important. If the examiners have thought it necessary to provide an introduction, it

will be because they think it will help you to understand the passage more easily.

The questions will test your ability to understand the writer’s ideas, to analyse the

writer’s techniques, and to evaluate the effectiveness of the writing. The total number

of marks available is 30, with the number of questions varying and the number of marks

allocated to each question being shown at the end of each question. It is important to

use your time wisely so that you answer all the questions and are not rushing to finish

the last two or three questions!

The number of marks allocated to a question will give you a clear idea of the number

of points required. A question for 1 mark can probably be answered in very few words,

while a 3 or 4 mark question (especially if it is coded A or E) will require a detailed

answer making a number of points. A common mistake is to spend too much time

on the early questions. Remember that the questions at the end may be quite ‘high

value’ ones – so it’s important to give them enough time. Also, don’t waste time writing

unnecessarily long answers with introductions which simply repeat the question

– get to the point quickly. Remember to look at the code letter(s) for the question

and focus your answer appropriately.

As well as an indication of the number of marks allocated, there is a code letter to tell

you which skill is being tested in each question. These codes are: U for Understanding,

A for Analysis, E for Evaluation. Sometimes these are combined to indicate that there is

a focus on more than one skill – for example, U/E indicates that you are being asked to

show an understanding of the writer’s ideas and to make an evaluation of them.

The importance of reading

The best preparation for this part of the examination is extensive reading of the types

of English from which the passages are usually selected (ie non-fiction). This should be

done over a period of time before your exam – you cannot expect to become familiar

with this type of complicated writing by looking at a couple of past papers. The more

comfortable you become with the type of writing, the less daunting the passage in the

exam will seem. You may even begin to guess the types of questions the examiners will

ask.

Here is a useful homework (or extra study) exercise. Each week pick one an article from a

good quality newspaper. Read the article and try to answer these three questions:

1 What are the writer’s main ideas?2 Are there any interesting language features? (e.g. metaphors, imagery, etc). Write down the language feature and try to comment on it. Say what effect it has.3 Can you find any words that you don’t know the meaning of? If so, write them down in a notebook, and look up the meanings.

Regular reading of non-fiction, newspapers, magazines and periodicals should provide

you with the quality of reading experience required at this level. The quality of the

writing is the main criterion in passage selection and your teacher should be able to

guide you towards appropriate materials. Looking at previous exam papers is the

most obvious way of making yourself familiar with the layout of the paper and the style

of questioning within it. Recent past papers in Intermediate 1 and Intermediate 2

English are available, published by Bright Red Publishing, and can be purchased in

most bookshops!

Some specific advice

For questions on understanding, you should answer these as far as possible ‘in your

own words’. This means that you have to demonstrate that you understand the more

complex words and phrases used in the passage. If you simply quote or use the words

already in the passage, the marker won’t know whether you understand what they mean

– and will quite reasonably assume that you don’t. The number of marks allocated to an

Understanding question will clearly indicate the number of points you are expected to

make. Try to make your answers to these questions fairly brief.

For example, in the 2010 SQA Intermediate 2 Close Reading paper (about an exhibition

on a Chinese Emperor), Question 3 asks candidates to look at lines 6 -10, which were:

“That probably goes for the vast majority of people in the West. And given that he is

one of the most colossal figures ever to have walked the earth, that is rather shocking.

For Qin Shihuangdi, its first Emperor, created China more than two millennia ago,

~establishing the world’s longest-lasting empire. A visionary, a brutal tyrant and a

megalomaniac, he is the greatest historical figure that most of us have never heard of.”

The question asks: “Give in your own words two reasons why it is “rather shocking” that

most people in the West do not know about Qin.”

Notice two things about this question: first of all the question makes it very clear that

you have to use your own words in your answer. This instruction is given in bold to

show how important this is. You will notice that all Understanding questions of this type

at both Intermediate 1 and Intermediate 2 levels have this instruction in bold, where

it is required. The most common way to lose marks in Understanding questions at

Intermediate 1 and 2 Close Reading is by copying words from the passage.

To answer this question, you need to try to change key expressions from this paragraph

into your own words:

For “created” you could write that Qin set up or founded China.

For “colossal figure” you could write that he is a very important person in history.

For “long-lasting” you could write that his regime was the most permanent, etc.

Note that, in this case, “greatest” does not mean “largest.”

Link questions are common, although not asked every year. You must demonstrate an

understanding of each of the two paragraphs (or sections) being linked. In addition you

must identify the word or words in the link sentence which connect with the preceding

paragraph and the word or words in the link sentence which connect with what follows.

So there are four elements in a successful answer:

. a reference to or quotation from the link sentence which refers to the idea(s) of the preceding paragraph or section. an understanding of the idea(s) of the preceding paragraph or section. a reference to or quotation from the link sentence which refers to the idea(s) of the coming paragraph or section. an understanding of the idea(s) of the coming paragraph or section

Note however, that a full understanding of all four elements is not essential to gain full

marks for this question. For Intermediate you would need to deal with the first and

third bullet points, but for Higher you would need all four.

For example, Question 10 in the 2010 SQA Intermediate 2 Close Reading paper asks

you to: “Explain how the sentence ‘But for the First Emperor establishing complete control

over his empire was not enough.” (line 56) works as a link between paragraphs at this

point.” Notice that the number in brackets tells you exactly where in the passage to find

your answer. To be given full marks at Intermediate 2, you would have to identify which

parts of the sentence point back to what the writer has just said, and which parts of the

sentence point forward to what he is about to say. Your answer could be something like

this: “establishing complete control of his empire” refers back to what he has just been

talking about, and “was not enough” points forward to what he is going on to say.

You would also be awarded one mark for writing that the word “But” introduces a

change of direction in the writer’s argument.

Questions on imagery and on word choice are questions most English candidates

find especially difficult. It’s not easy to ‘learn’ how to do them, since your ability here

depends on your sensitivity to language, and this is something that has been growing

gradually since you started learning to read.

The following bits of advice, however, might help:

. Some marks may be available at Intermediate 1 for quoting a word or identifying an image, but for Intermediate 2, more marks are usually available for the ‘quality of comment’. The comment must be specific to the word or image being asked about and how effective is it in adding to the writer’s meaning. When answering on word choice, try to go beyond what a word means, and explore what it suggests (in technical terms: connotation rather than denotation). When answering on imagery, try to show how the literal root or origin of the image is being used by the writer to express an idea in a metaphorical way

Page 12: PAST PAPER QUESTIONS FOR MAY EXAMS - Elgin · PDF filePAST PAPER QUESTIONS FOR MAY EXAMS Of cial SQA past paper questions for Maths, Biology, Physics and Chemistry - from Standard

12 | Official SQA Past Papers Supplement - English

Looking at word choice, Question 8 from the 2010 SQA Intermediate 2 Close Reading paper asks candidates “What does the writer gain by using the word ‘toil’

(line 38) rather than the word ‘work’? If you look at the sentence from which the

word is taken you get a clear sense that the work involved is no ordinary work:

“One of the most miserable punishments, which very often proved to be a death

sentence, was to be dispatched into the wilderness to toil on the construction of

the wall Qin Shihuangdi had ordered to be built along the northern frontier of the

empire.”

The word “toil” adds to the sense conveyed by “miserable punishments” and

“very often proved to be a death sentence.” It suggests that the work means real

hardship or drudgery.

Questions on sentence structure are also quite demanding. As with questions on imagery

and word choice, it’s not easy to ‘learn’ how to answer them. You have to be able to

recognise relevant features of sentence structure (eg brevity, length, use of listing,

climax, anti-climax, repetition, use of questions, balance, period), and marks are also

given for your ability to comment on their effect in context.

Question 4 from the 2011 SQA Intermediate 2 Close Reading paper asks candidates

to “Show how the writer’s word choice or structure helps to reinforce this point (about

attitudes to texting)” You are directed to lines 11-13 which read:

“But has there ever been a linguistic phenomenon that has aroused such curiosity,

suspicion, fear, confusion, antagonism, fascination, excitement and enthusiasm all at

once as texting?”

Here the writer makes it clear that there are many different attitudes to texting. If you

concentrate on structure, he emphasises this by listing the different attitudes, or by

using a question. In this case it is a rhetorical question inviting you to agree with him.

Questions on tone are possibly the most difficult area of all. You may have to identify

the writer’s tone at a particular point in the passage (eg anger, contempt, regret,

nostalgia, irony, humour) and you also may have to explain how the writer establishes

the tone. The ‘how’ part is often done best by exploring other aspects of language such

as sentence structure, imagery, and word choice (see the sections above) since these are

often used to convey tone. Also, features such as sound, exaggeration and anti-climax are

often used to establish tone. Occasionally, you are given the writer’s tone and asked how

the writer has used language to create it.

Question 11 of the 2010 SQA Intermediate 2 paper asks candidates to: “Show fully

how the writer introduces a tone of doubt when he writes about the prospects for

opening the tomb (lines 60-62)”

Lines 60-62 read: “The tomb itself may never be opened because of the sensitivities of

disturbing the Emperor, although some archaeologists hope that improved technology

may one day allow some form of exploration.”

Here a sense of doubt, or a tone of doubt, is created by the use of words such as

“may,” “hope,” ‘one day.” It is also suggested by the use of the expressions “some

archaeologists,” and “some form.” In your answer, if you picked out two of these

examples you would be given two marks. You would also be given two marks for one

example plus a comment explaining how your chosen example creates a tone of doubt.

The need for two examples, or one example plus an explanation, is indicated by the

word fully in the question: “Show fully...”

Sometimes a question simply asks you to show how ‘the writer’s language’ does

something or other. This means you’re not being guided towards a specific technique

such as sentence structure or tone. For these questions you must find the most

appropriate technique(s) and then deal with it/them in the way suggested above.

Remember, however, there will usually be no marks for simply identifying a feature or

quoting a word or image. As a starting point, the most straightforward technique to

look for is word choice. Answer the question by stating that you are going to look at

word choice, then write down the example of interesting word choice which you have

selected, then add your comment on the meaning/effect of the words selected.

THE CRITICAL ESSAY

The Critical Essay exam paper at Intermediate 1 lasts for forty-five minutes and, at

Intermediate 2, it is ninety minutes long. Both exam papers will have a range of essay

questions on different genres of literature, film and TV drama, and the study of

language. The questions will be arranged in five sections and you must answer any one

question from any section at Intermediate 1 and two questions from different sections

at Intermediate 2. The questions will test your ability to select from your knowledge of

a text (and the techniques used in its construction) in order to write a relevant response

to the chosen question. At Intermediate 1 and 2 the essays are marked out of 25

(making the total for the paper at Intermediate 2 50 marks).

The five sections are as follows:

Section A - Drama - usually a choice of two questions at Intermediate 1

and three questions at Intermediate 2

Section B - Prose - usually a choice of two questions at Intermediate 1

and three questions at Intermediate 2, covering novel, short story and non-fiction

Section C - Poetry - usually a choice of two questions at Intermediate 1

and three questions at Intermediate 2

Section D - Film and TV Drama - usually a choice of two questions

at Intermediate 1 and three questions at Intermediate 2

Section E - Language - usually a choice of two questions at Intermediate 1

and three questions at Intermediate 2

It is important to allocate your time sensibly. At Intermediate 1, you have 45 minutes to

write an essay and must make sure that your finish it in the time given. At Intermediate

2, you have approximately 45 minutes for each essay. If you spend a lot longer on one

essay, you may gain an extra mark or two, but a very short second essay is likely to score

very few marks.

If you look at previous past papers, you will see that all the questions are structured in a

very similar way. There are two sentences:

The first sentence provides the initial focus or ‘gateway’. If the text you want to write

about does not meet the restriction in this part, then you are not going to be able to write

a relevant essay and you will not pass. If the text does fit, then you might be able to go on

to write a suitable essay. However, this is not guaranteed, because you must be able to

deal with the requirements in the next sentence.

The second sentence is the one that provides the key instruction for what you have to do, and your essay will be judged on how successfully you handle this part. You must

not think that anything you write will automatically be relevant just because the text

fits the definition in the first sentence. You must do exactly what is asked for in the

question. Notice that the question may contain more than one instruction and that you

must address the whole question. This is very important. Many people do not do well

because they don’t fully address the second sentence in the question.

For example, Question 1 from the 2010 Intermediate 2 Critical Essay paper asks:

“Choose a play in which a central character feels increasingly isolated from

those around her or him. Explain why the character finds herself or himself isolated,

and show what the consequences are for the character concerned.”

In answering this question many candidates concentrate on listing the reasons for the

main character becoming isolated in the play: it is important to devote a significant

section of your essay to the consequences for the main character of that isolation.

Question 4 from the 2010 Intermediate 2 Question paper reads:

“Choose a novel or a short story which gives you an insight into an aspect of human

nature or behaviour. State what the aspect is, and show how the characters’ actions

and relationships lead you to a deeper understanding of human nature or behaviour.”

In addition to making clear what aspect of human behaviour the novel or short story

is concerned with, your answer to this question must include your thoughts on how

the novel or short story has helped you to a better or deeper understanding of human

behaviour. In other words, you must try to write about theme. Ideally, you should

try to refer to theme throughout your essay, as well as in your conclusion.

Page 13: PAST PAPER QUESTIONS FOR MAY EXAMS - Elgin · PDF filePAST PAPER QUESTIONS FOR MAY EXAMS Of cial SQA past paper questions for Maths, Biology, Physics and Chemistry - from Standard

Official SQA Past Papers Supplement - English | 13

Use SCOT2 at www.brightredpublishing.co.uk for 30% off and free post and packaging!

Above all else, try to write a relevant essay. This means you are unlikely to be able to

write everything you might want to say, but it’s much better to write a short essay which

is clearly relevant than a long essay which covers everything you know and ignores the

question. The former is likely to pass; the latter will probably fail. Remember to keep

the key words of the question in mind while planning your essay.

For example, Question 7 of the SQA 2010 Intermediate 2 Critical Essay paper asks:

“Choose a poem which could be considered as having a powerful message.

Show how the poet effectively conveys this message through his or her poetic

techniques.”

In this question it is important that you identify (right at the start of your essay) what

the important message is. Keep referring to this message throughout your essay, and

provide evidence from the poem to show how the poet puts the message across.

At Intermediate 1 and Intermediate 2, it’s advisable to have at least one ‘back up’ text

just in case. However, depth of preparation is every bit as important as the number

of texts prepared. For example: if you prepare a suitable novel and are able to write

confidently about such areas as theme, characterisation (of one or two main characters

and of two minor characters), setting (in time and place), key incidents (including the

opening and the conclusion), narrative technique, structure, symbolism, then it is highly

unlikely that you will be stuck for a question; whereas if you prepare the same novel but

are able to write about, for example, only one character, then you are very likely to

have difficulty.

You should return again and again to reading and studying your chosen texts. Make

notes; add to your existing notes. Learn from your successes and failures in previous

essays, but never, under any circumstances, learn a previous essay by heart, no matter

how good a mark it was given – it was answering one particular question; the question

in the exam will be different. The secret is to have plenty to say and then to select from

that in order to construct a relevant essay. Looking at previous exam papers will allow

you to see examples of the types of question which will be asked!

Some specific advice

Some allowance is made for the fact that because this is an examination you are writing

under pressure. It is recognised that you might make one or two careless slips and that

you don’t have time to redraft your work. Nevertheless, if your writing is not sufficiently

accurate to meet the Performance Criterion for Technical Accuracy, you will not pass.

Common errors to be avoided are: failure to start a new sentence when required

(especially using a comma when a full stop is needed), misspelling of common words,

misuse of the apostrophe, confusion of ‘done/did’, ‘gone/went’ etc, and using slang or

colloquial language.

If you look at the list of Performance Criteria for Critical Essay in Intermediate 1 and

Intermediate 2 English you will see that in ‘Understanding’ it talks about the ‘main

point(s)... of the text(s)’ at Intermediate 1 and the ‘central concerns … of the text(s) at

Intermediate 2.

This is of great importance. You must demonstrate to the Marker that you have a firm

grasp of what the text as a whole is about. This means knowing not just what simply

happens in a novel or a play or what the content of each line of a poem is. Every

worthwhile text in English has an overall idea which it is exploring, and your personal

understanding of this (relevant to the question you are answering) must be a key

element in your essay.

Analysis

At Intermediate 1, The response describes some of the more obvious ways in which aspects

of structure/style/language contribute to meaning/effect/impact.

And at Intermediate 2, The response explains in some detail ways in which aspects of

structure/style/language contribute to meaning/effect/impact.

One of the Performance Criteria requires that you deal with ‘aspects of structure/

style/ language’ and how these ‘contribute to meaning/effect/impact’. It is, therefore,

important that you learn about the techniques used by the writers of the texts you study.

Reference to these techniques, however, is only of value if it supports the line of thought

in your essay. Read carefully the advice and information given in the past examination

papers - at the start of the paper and in the boxes in the sections. You should not deal

with techniques in isolation, and you should not structure your essay around them.

Evaluation

At Intermediate 1, The response contains a stated or implied personal reaction to the

content or style of the text, supported by some relevant textual reference.

At Intermediate 2, The response reveals engagement with the text(s) or aspects of the

text(s) and stated or implied evaluation of effectiveness, substantiated by some relevant

evidence from the text(s).

It is important that you can write about how well the writer has engaged your interest

both intellectually and emotionally. You should be able to evaluate and explain this

response in terms of the features of the genre studied and the relative success achieved

by the writer in using them.

The study of a short story is as valid and as valuable as the study of a novel. It should

not, however, be thought of as an easier option just because it is shorter. Writers of short

stories employ specific techniques associated with the genre, and it is fair to say that,

because of the very specialised nature of this genre, writing well about a short story can

actually be harder than writing about a novel.

In the prose section, as well as questions on the novel and the short story, there are

questions on non-fiction. Questions on non-fiction usually appear as Question 6 in the

Critical Essay paper. If, deliberately or by accident, you answer any of these questions

using a novel or a short story, your script will be referred to the Principal Assessor, who

will apply an appropriate penalty. This penalty could be the difference between your

passing and failing the exam, and so you should check carefully that the text you are

writing about is entirely suitable for the question. The study of quality non-fiction is

as valid and valuable as the study of prose fiction. Such works include: biography and

autobiography; travel writing; essays and works on history, politics, current affairs,

media issues, science and technology, religion, environmental issues, etc. Essays on

non-fiction are judged in the same way as all others, but candidates should be aware

that many of the techniques used in prose non-fiction are different from those used in

prose fiction.

In essays on poetry there are two common faults which you should try to avoid.

1. You don’t have to work through a poem line-by-line. While you mustn’t ignore

significant sections of the poem or distort its overall idea (see the section on ‘the

central concerns’ above), your essay should be shaped to answer the question

relevantly and not dominated by an insistence on examining every line in order.

2. While understanding and appreciation of poetic techniques are vital in a good essay

on poetry, an exhaustive list of all the techniques used in a particular poem is not very

helpful. Remember about the overall idea (see the section on ‘the central concerns’

above) and remember that techniques enhance the overall impact of a poem – they do

not have a life in themselves.

Questions on film and TV should be approached in exactly the same way as questions

on drama or prose or poetry. The questions are structured the same way, and the

warnings given above about relevance and ‘central concerns’ are just as important here.

Similarly, specialised techniques such as camera angles, lighting, soundtrack and special

effects have their place in an essay on film or TV drama, but only if your comments

on them are relevant to the question and support your understanding of the text as a

whole. Note that the term ‘TV drama’ refers to a single play or a series or a serial.

The questions on Language are included for candidates who have made a specific

study of the subject . These questions are not asking for general essays, no matter how

much you think you know about the topic. They are judged against the same criteria of

relevance, knowledge, analysis, use of evidence, evaluation and quality of writing as all

other essays.

Page 14: PAST PAPER QUESTIONS FOR MAY EXAMS - Elgin · PDF filePAST PAPER QUESTIONS FOR MAY EXAMS Of cial SQA past paper questions for Maths, Biology, Physics and Chemistry - from Standard

14 | Official SQA Past Papers Supplement - English

Bright Red Official Past Papers Intermediate 1 English

ISBN: 978-1-84948-188-5£8.99

Bright Red Official Past Papers Intermediate 2 English

ISBN: 978-1-84948-197-7£8.99

FINAL ADVICE

Close Reading

. Read the passage as carefully as you can in the time that you have. Try not to panic if you find it hard to take in all the meaning quickly: the questions will take you through the passage stage by stage. Pay attention to the introduction at the start of the passage - this helps to give you a general sense what it is about

. Use your own words when asked to do so - not using own words is the most common way to lose marks

. When a question contains words such as “show”, “explain,” or “show/explain fully” remember that explanation or comment is required from you

. The number of marks should give you an idea of how much you have to write (or how many points you should make)

Critical Essay

. Remember to read all the questions in order to find ones

that suit you best

. Be careful to choose from the correct sections. In other

words, don’t write about a poem in response to a question from the Prose section (even if the question seems to fit!)

. Refer to the text you have chosen, its author, and the key

words of the question in the first paragraph of your essay. (Try to refer to themes too)

. Remember to deal with all the demands of the second

line of the question

. Keep your essay relevant throughout. Try to maintain

a line of thought (an answer to the question)

. Round your essay off with a suitable conclusion which,

once again, refers back to the key words of the question

. Make sure your essay is reasonably accurate - check for

mistakes at the end, and correct them if you have time

Page 15: PAST PAPER QUESTIONS FOR MAY EXAMS - Elgin · PDF filePAST PAPER QUESTIONS FOR MAY EXAMS Of cial SQA past paper questions for Maths, Biology, Physics and Chemistry - from Standard

Official SQA Past Papers Supplement - English | 15

Use SCOT2 at www.brightredpublishing.co.uk for 30% off and free post and packaging!

HIGHER

ENGLISHAbout the author

Christopher Nicol is a Chartered Teacher in the English Department of Galashiels Academy. He holds a PhD in Applied Linguistics and has taught in the secondary and university systems of Scotland, England, France and America.

Highers: the last lap

With the prelims just a fading memory and May 17th looming ever larger, it’s getting increasingly important to deal with those lingering weaknesses that the prelims helped pinpoint. Yes, it is not the best of times to be saying this, since your Folio at the moment will be absorbing a lot of your time, too. But in the remaining weeks, you can do a lot to make sure that your 25 in Close Reading jumps to nearer 35 and that your 17 for that ‘Hamlet’ essay makes it to nearer 21. How? Your teacher will be working flat out to help you make that leap forward, but there’s a great deal you can do yourself at home to make that final advance. In the next few pages we’ll be adding to that help by taking a calm look at what is being expected of you and how best to ensure the result you need for the next step in your academic career.

Provided you are prepared to organise your time in a regular work routine, you can start right now sizing up the hurdles ahead - and how best to get over them. The next sections will offer you practical advice and tips to help you better understand what examiners may be looking for. So let’s get started.

Page 16: PAST PAPER QUESTIONS FOR MAY EXAMS - Elgin · PDF filePAST PAPER QUESTIONS FOR MAY EXAMS Of cial SQA past paper questions for Maths, Biology, Physics and Chemistry - from Standard

16 | Official SQA Past Papers Supplement - English

Close Reading: how to better your prelim performance

All year, your English teacher - and guides like this one – have been advising you to

familiarise yourself with the kind of texts Close Reading exams are based on. But admit

it, were you as dedicated to the task as you might have been? Hmm? If not, it’s certainly

not too late to rectify matters. In English, the Close Reading passages will probably come

from the ‘Comment’ or ‘Opinion’ columns of quality newspapers where experienced

writers engage with ideas and topics of current interest. Get online today and start

getting to grips with the kind of vocabulary and structure you may expect to encounter.

You need to start right now familiarising yourself with the styles of such writers, their

vocabulary and their way of structuring their pieces.

Most quality newspapers and journals will allow you free access to their web-sites. In

the few weeks remaining to you, make sure you regularly pick up or view online quality

papers such as ‘The Scotsman’ or the English ‘heavyweight’ papers. Regular reading

of such articles is the only way to increase your ability to read and understand them at

speed. And the ability to read and re-read at speed is of the utmost importance in this

kind of exam – as we are about to see.

From your experience in the prelims, you know the drill: the Close Reading exam paper

lasts for one hour and forty-five minutes and will have two passages on a related theme.

The passages are selected from works of non-fiction from the kinds of newspapers

and journals we have just been discussing. The ideas will be complex and expressed

in sophisticated, carefully structured English. The total length will be in the region of

1,500 words and the length of each of the two passages varies from year to year: the first

passage may be longer than the second, or the first passage may be shorter than the

second, or both passages may be of similar length.

There is usually a brief introduction (printed in italics) just before each passage begins.

This can be very important. If the examiners have thought it necessary to provide an

introduction, it will be because they think it will help you to understand the passages

more easily.

To this end, some teachers will also suggest you immediately read the first paragraph

and then the final one to see where the piece is going. If you find this kind of reading

difficult to take in at a first reading, another good tip is to read the first sentence of each

paragraph to help find a way through what can be quite dense text.

The questions will test your ability to understand the writers’ ideas, to analyse the

writers’ techniques, and to evaluate the effectiveness of the writing. There will always be

at least one question requiring comparison of the passages. The total number of marks

available is 50, with the number of questions varying. The number of marks allocated

to each question is shown at the end of each question. As well as an indication of the

number of marks allocated, there is a code letter to tell you which skill is being tested in

each question. These codes are: U for Understanding, A for Analysis, E for Evaluation.

Sometimes these are combined to indicate that there is a focus on more than one skill –

for example, U/E indicates that you are being asked to show an understanding of the

writer’s ideas and to make an evaluation of them.

Pacing for success

One hour and forty five minutes might seem a long time, but you will be amazed how

quickly it goes. You cannot allow yourself to sit and fret over a question which has you

bamboozled. Move on, leave space and if there is time at the end, you can come back to

it. Pace yourself so that in every ten minute period you have targeted, let’s say, 5 marks.

And above all else, make sure you read how much each question is worth. If you’ve spent

five minutes and ten lines on a question worth 1 mark, there’s something far wrong.

Similarly, if you have dashed off a line or so for a question worth four marks, then you’re

throwing marks away.

Check, too, on any high value questions (often towards the end) and make sure you have

left sufficient time for them. The high value questions (usually of the E or evaluation

kind) will probably require you to write in ‘mini-essay’ form, but most other answers do

not be in sentences. And if a question has an A or an E beside it, you will have to have at

least one (and probably more than one) quotation from the text in inverted commas.

Above all, don’t waste time repeating the wording of the question in your answer, plunge

in with the evidence required. For instance, in word choice questions, simply nail the

word you have selected and give your comment e.g. If you were being asked about

the uncertainty with which someone entered a room and the word you have selected

to comment on is ‘sidled’, you would simply write ‘sidled’- suggests the writer felt

embarrassed or guilty, so enters in a tentative manner. Your primary teacher might have

given you hell for not answering in full sentences, but unnecessary wording steals time

you don’t have in Close Reading exams.

Improving Understanding questions

So let’s see how you should go about answering the kind of questions that might be

losing you marks unnecessarily. In ‘Understanding’ questions (marked U), it should be

taken for granted by this time that these must be answered in your own words. (Even

though the individual questions do not state this, there is a clear instruction on the front

cover of the exam paper, and this is repeated at the beginning of the questions.) Why?

Because if you ‘lift’ a word or phrase from the text, you are not showing the examiner

you know what it means – which is the whole point of the exercise in the first place!

Let’s take an example: suppose the answer revolved around understanding

As for their parents, there was a distinct lack of compatibility.

It is pointless, therefore, including the word ‘compatibility’ or ‘compatible’ anywhere in

your answer since the examiners are fishing to know if you know what that word means.

So you would need to answer along the lines of

Their parents had trouble getting on with each other.

Or

They had very little in common with each other.

Answers to ‘Understanding’ questions can often be quite brief and bullet points will

often be sufficient.

A question which often elicits groans from most candidates is the ‘link’ question.

It is a common question although it’s not asked every year. Note that this is an

‘Understanding’ question. You must demonstrate an understanding of each of the two paragraphs (or sections) being linked. So how do we go about it? You need to include

four elements in your answer:

. a quotation (from the link sentence) which refers to the idea(s) of the preceding paragraph

. an understanding of the idea(s) of the preceding paragraph

. a quotation (from the link sentence) which refers to the idea(s) of the coming paragraph

. an understanding of the idea(s) of the coming paragraph.

So what does that look like on paper? Suppose the link sentence was

These happy years had not prepared her for the hell of prison life.

Let’s say the previous paragraph had described certain happy years ‘she’ had spent

somewhere and the coming one dealt with the horrors ‘she’ faced in prison. You would

need to produce an answer of four elements something like this:

These happy years [Element 1] looks back to the previous paragraph in which we learn

she had enjoyed many carefree childhood pleasures [Element 2]

the hell of prison life [Element 3] looks forward to the content of the rest of the paragraph

which details the filth and squalor encountered behind bars. [Element 4]

But linking questions are not the only Understanding questions where valuable marks

can be lost and time wasted.

Let’s look at how that might appear in the exam.

Page 17: PAST PAPER QUESTIONS FOR MAY EXAMS - Elgin · PDF filePAST PAPER QUESTIONS FOR MAY EXAMS Of cial SQA past paper questions for Maths, Biology, Physics and Chemistry - from Standard

Official SQA Past Papers Supplement - English | 17

Use SCOT2 at www.brightredpublishing.co.uk for 30% off and free post and packaging!

Here is a link question from the SQA’s Higher English in 2008.

Example – 2008, Close Reading Question 3 (a)

15 Then there is the proliferation of action groups dedicated to stopping construction of roads, airports, railway lines, factories, shopping centres and houses in rural areas, while multifarious organisations have become accustomed to expending their time and energies in monitoring and reporting on the state of grassland, water, trees, moorlands, uplands, lowlands, birds’ eggs, wildflowers, badgers, historical sites and countless other aspects of the landscape and its inhabitants.21 It might be thought—indeed, it is widely assumed—that it must be good for the countryside to be returned to the central position it enjoyed in British life long ago. Yet there is a particularly worrying aspect of the new rural mania that24 suggests it might finally do the countryside more harm than good. This is the identification, in the current clamour, of the countryside in general and the landscape in particular with the past—the insistence on the part of those who claim to have the best intentions of ruralism at heart that their aim is to protect what they glibly refer to as “our heritage”. This wildly over-used term is seriously misleading, not least because nobody appears ever to have asked what it means.

3. (a) By referring to specific words or phrases, show how lines 21–24 perform alinking function at this stage in the writer’s argument.

Comment: Now, by looking at the lines in question we see that the ‘looking back’ is to

a time when the countryside was left untouched, and the looking forward is to a worry

of the writer’s: that a bogus argument is about to be put forward to retrieve that earlier

position. So what are our ‘looking back’ and ‘looking forward’ phrases?

Answer: returned to the central position [Element 1] looks back to aims of the action

groups to halt countryside –destroying construction projects [Element 2]

a particularly worrying aspect [Element 3] points forward to concerns the writer has

about how ‘heritage’ is being claimed as a reason for protecting the countryside.

[Element 4]

But linking questions are not the only Understanding questions where valuable marks

can be lost and time wasted. Questions in which you are asked to ‘summarise’ or to

‘identify the main points’ or to give the ‘key reasons’ often plunge candidates into

time-wasting long-winded answers. Don’t include any of the supporting evidence or

examples the writer uses – these will weaken a ‘summary’. Identify the main idea; do not

even try to cover all the detail the writer gave; that is the very opposite of a summary.

Mastering Analysis questions

In a moment, we’ll look in more detail at some of the various questions that crop up

with an A for analysis tagged on to them. But here’s something you really must bear in

mind before tackling any Analysis question. You don’t get any brownie points just for

spotting a metaphor or inversion in a sentence or a particular image. Spotting a device

in question must be followed up by suggesting what the effect of that is on the reader.

In other words, why has the writer used that particular choice or order of words; what is

it adding to your reading experience? Phrases like This helps us share his anger about…

or The metaphor of…. helps us see her wonder at…. The inversion on the word ‘dutifully’

makes it clear that the writer feels that……..will help get across to the examiner what you

feel the effect of this particular choice of language.

Now let’s look at imagery questions. They don’t need to be the hurdle that some

candidates imagine them to be. Go first to what English teachers like to call the ‘root’

image. By ‘root’ image they mean the object that something is being compared to. For

example, if you encountered, say,

Refugees from the war-zone flooded into the camps.

and you were asked to comment on the effectiveness of the image, you would first

have to think about all the ideas you associate with floods: powerful, unstoppable,

overwhelming, covering everything in their path. Then try this formula: Just as… so, too,

was… So, we arrive at:

Just as a flood is an unstoppable torrent, so too was

the wave of refugees that swept into the camps.

Now follow this up with what our old friend the ‘effect’ of all this was:

This metaphor effectively underlines the overwhelming nature of

the sheer number of refugees that came pouring into the camps.

Next time you are faced with an imagery question, try the Just as…, so, too,…formula.

You’ll find it can help a lot. Here is an imagery question to try:

Example – 2011, Close Reading Question 2

15 The most powerful example of this trend is found in the world of video games. And the first and last thing that should be said about the experience of playing today’s video games, the thing you almost never hear, is that games are fiendishly, sometimes maddeningly, hard. The dirty little secret of gaming is how much time you spend not having fun. You may be frustrated; you may be confused or disorientated; you may be stuck. But when you put the game down and move back into the real world, you may find yourself mentally working through the22 problem you have been wrestling with, as though you were worrying a loose tooth.

2. “...how much time you spend not having fun.” (lines 18-19) Show how, in lines 15–22, the writer conveys the difficulty of playing videogames by his use of imagery.

Comment: Now, you need to be sure of your 2 marks. Typically, examiners are invited

to give 2 marks for a particularly good answer and 1 each for answers that are not so

full. This is not a question that will take a long time to answer, so it might be a good

idea always to offer two in these circumstances. Imagery worth looking at might be

‘stuck’, ‘wrestling’ or ‘worrying a loose tooth’. Let’s take ‘wrestling’ as one example

of a possible answer.

Answer: wrestling: just as wrestling involves a close, tiring physical struggle with an

adversary determined to win, so too does your bout with the video game pit you against

an opponent seemingly equally determined to win.

Similarly troublesome can be the word choice question. It is like the imagery question

in that you need to think of not only what the word means (denotation) but what you

associate with this particular word (connotation). So, if you’ve picked out the word

skinny you know that it means ‘thin’. But while most women would like to be called

‘thin’ or ‘slender’, they would jib at ‘skinny’? Why? Because it has associations of being

unattractively underweight with bones sticking out in strange places. So how do we get

this down on your answer paper? Good news: word choice questions can be answered

quickly. Simply write down the selected word in inverted commas and add what it

suggests to you: ‘skinny’ – suggests the model is unattractively underweight in the eyes

of the writer. No long sentences, just the targeted word and a brief comment of what its

selection suggests to you.

Let’s try that with a word choice question from SQA’s 2010 Higher English paper

Example – 2010, Close Reading Question 1 (b)

In a world changing faster now than ever before, the dispossessed and the ambitious are flooding into cities swollen out of all recognition. Poor cities are struggling to cope.Rich cities are reconfiguring themselves at breakneck speed. China has created anindustrial powerhouse from what were fishing villages in the 1970s. Lagos and Dhakaattract a thousand new arrivals every day. In Britain, central London’s population hasstarted to grow again after 50 years of decline.

1. (b) Show how any two examples of word choice in this paragraph emphasise the impact of the growth of cities.

Comment: Happily, there is no shortage of choice here. You might see possible

candidates in ‘flooding in’, ‘out of all recognition’, ‘struggling’, ‘breakneck speed’,

‘industrial powerhouse’ to name just a few. Which ones offer the best opportunity for

discussion? A good, straightforward choice might be ‘flooding in’ and ‘at breakneck

speed’. Let’s try them.

Answer: flooding in: just as a flood suggests a huge, unstoppable force, so too do the

people who surge into ever-growing cities suggest a wave of incomers impossible to

control or hold back. breakneck speed: suggests change to the city size is fast to the

point of being life-threatening and dangerous.

Page 18: PAST PAPER QUESTIONS FOR MAY EXAMS - Elgin · PDF filePAST PAPER QUESTIONS FOR MAY EXAMS Of cial SQA past paper questions for Maths, Biology, Physics and Chemistry - from Standard

18 | Official SQA Past Papers Supplement - English

Sentence structure, too, can cause concern among candidates. And while sentences

can be manipulated to give almost any effect in literature, there is, happily, a limit to

the things that sentences tend to do in the Higher English Close Reading exam; these

should be memorised long before you go into the exam room. Know the commoner

punctuation marks and their purposes and effects in a sentence; the effects created

by unusually long or short sentences; the use and effects of climax or anti-climax in

a sentence; how the repetition of particular words or phrases contributes to a certain

effect; the effect which unusual word order in a sentence creates; and if all else fails,

there is always our old friend from Standard Grade – the list!

The trick is to spot which of these structuring devices you are observing at work. Run

through all of them in your mind if the answer is not immediately obvious. The guides to

passing Higher English are pretty thorough in detailing the twists and turns of sentence

structure. Get acquainted with one and make absolutely sure above all, as in imagery

or word choice questions, you discuss the effect this particular manifestation of the

sentence structure is having on you, the reader.

For examples of this type of question, look at question 9 in the 2008 paper, questions 1(b) and 10 in the 2006 paper, or question 6(b) in the 2004 paper.

But for many candidates, tone questions are the ones that give the most bother. If you

are not familiar with the various tones journalists play around with in ‘Comment’ or

‘Opinion’ pieces you may be a bit at sea. All the more reason, therefore, as suggested

earlier, to get into daily contact with writing of this kind in quality newspapers and

journals. It gets easier the more familiar you become.

Not only will you have to identify the writer’s tone at a particular point in the passage

(e.g. angry, contemptuous, regretful, nostalgic, ironic, humorous), you’ll also have

to explain how the writer establishes the tone. The ‘how’ part is often done best by

exploring other aspects of language such as sentence structure, imagery, and word

choice (see the sections above) since these are often used to convey tone. Also, features

such as sound, exaggeration and anti-climax are often used to establish tone. For an

example of this type of question, look at question 7(b) in the 2008 paper.

There will always be at least one question at the end of the Close Reading paper

requiring some comparison of the two passages. From 2011/12 it will ask you to compare

the similarities and /or differences in the key ideas in both passages. You will always

have to make reference to both passages but you don’t have to give them both the same

amount of attention. You can answer the question either by writing an answer or by

giving a series of developed bullet points.

For further details, look at the Revision of Comparison Question information on the

Higher English Announcements of the SQA website, where an example based on the

2011 Question paper is provided with accompanying Marking Instructions.

A good idea is to have a look at the comparison question(s) before you start so that while

you are working your way through the other questions and becoming more familiar with

the ideas in the passages, you will be able to give some thought to what you might say in

the comparison question(s).

Writing a better Critical Essay

As in the Close Reading paper, time is of essence. You have ninety minutes in total to

write two essays which might suggest you have 45 minutes to write each essay. In fact,

you don’t!

Think about it. You are faced with a paper of five sections: drama, prose, poetry, film and

drama and language; each section usually has four questions. You have to answer two

questions, each one taken from a different genre. That means you have to read through

the questions in your chosen genres, decide after some reflection which questions you

are best placed to answer and then plan your approach.

So your original 45 minutes are now seriously reduced. All this means that you need to

know your preferred texts backwards, so that you can summon up the information and

quotations you will need instantaneously. Yes, by all means study your revision notes

thoroughly, but your knowledge of your texts themselves must be sufficiently profound

to be able to cope with whatever angle the questions my take. The read-through with the

teacher is only a start. You will have to read your texts many times on your own to ensure

this degree of familiarity.

And never, never think of going into the exam room with a ‘prepared’ essay and

squeezing it to fit any question to which it might be loosely applied. Examiners see

through this right away. What matters most is an answer which is focused squarely on

the demands of the question. Relevance is all. Better a shorter relevant essay, than a

longer essay which is only marginally relevant to the wording of the question. This might

mean you come out of the exam room feeling a bit frustrated; you have only been able to

show off a fraction of the knowledge you possess. But provided it is a relevant fraction,

you will have nothing to fear.

Reading the questions

Questions come in a two-sentence structure. The first sentence provides the initial focus

or ‘gateway’. You will be able to tell right away if, say, your chosen prose work ‘fits’ this

question.

Example – 2010, Critical Essay Question 8

Choose a novel in which a character seeks to escape from the constraints of his or her

environment or situation.

Let’s say you have been reading ‘The Strange Case of Dr Jekyll and Mr Hyde’. Is there

a sound ‘fit’ between your text and this question? Yes, you’re thinking. Dr Jekyll

is constrained from indulging his evil impulses by having to keep up to keep his

respectable appearance as a successful doctor. But go easy. You can still go badly wrong.

What does the second sentence say? This is the one that provides the key instructions

for exactly what you have to do. Running off and writing all you know about Dr Jekyll’s

constraints would be a bad idea. Your essay will be judged by how well and relevantly

you respond to the wording of this second sentence:

Explain why the character feels the need to escape and show how his or her response to the

situation illuminates a central concern of the text.

Notice this second sentence is itself divided into two parts. Make sure you do not devote

all your time to ‘explaining’ and cobbling on a last minute sentence or two about how

this ‘illuminates a central concern’. Make sure you cover both parts adequately.

And since we’re talking about timing, make sure you don’t get so far ‘in’ to this essay

that you leave only twenty minutes for the following essay. Keep your eye on the time at

all times! It’s no good getting one brilliant mark in one essay and a single figure in the

remaining one.

Questions on Language are to be strictly avoided unless you have studied this with your

teacher; it is not a soft option for those having difficulty finding a suitable question in

drama, prose or poetry. Similarly, Film and TV Drama questions are for those candidates

who have made a study of this area; it is a very bad idea even to think about writing on a

film you saw on telly the night before.

Structuring your essay

Happily, there is no single way of constructing an essay. The demands of each question

will often determine the essay’s final shape, as will your own personal way of formulating

a prose text of your own. Provided you pursue a clear line of argument in a way that

responds to the demands of the question, you stand to do well.

What must come across to the examiner, however, is that you have understood what

your text is about. It must become clear – and here your introduction might be a useful

place to start– that you have understood the main ideas of your chosen text. You need

also to analyse how these ideas are demonstrated through the text and then go on to

evaluate their effect on you, the reader.

Is this beginning to sound familiar? Understanding, analysis and evaluation? Our old

friends from Close Reading, admittedly under a new garb. Often, you will make clear

your understanding in a topic sentence, or statement or series of statements:

Page 19: PAST PAPER QUESTIONS FOR MAY EXAMS - Elgin · PDF filePAST PAPER QUESTIONS FOR MAY EXAMS Of cial SQA past paper questions for Maths, Biology, Physics and Chemistry - from Standard

Official SQA Past Papers Supplement - English | 19

Use SCOT2 at www.brightredpublishing.co.uk for 30% off and free post and packaging!

To the world at large, Dr Jekyll presents himself as a pillar of the medical and legal

establishment, one whose grand residence symbolises that respectability. Yet Stevenson

hints almost from the first moment we meet him in his handsome abode that there is

more to him than meets the eye.

This will need to be promptly followed up by supporting analytical evidence.

He is after all, Dr Jekyll M.D, D.C.L, LL.D, F.R.S., a man with doctorates in both medicine

and civil law, (qualifications which might suggest he should uphold, not violate, ethical

values), a man whom we discover giving one of his ‘pleasant dinners to some five or six old

cronies, all intelligent reputable men’ in his house ‘which wore a great air of wealth and

comfort’. We also note, however, that he had ‘something of a slyish cast’ about his features

and we remember, too, his handsome house is connected to the laboratory at the rear

which ‘bore in every feature the marks of sordid negligence’. This, together with……….

You will then need to give some evaluation of this evidence which makes clear your

interpretation of it:

Clearly, Stevenson is signaling to the reader that this is a world in which appearances may

be deceptive, a world where………..

As you prepare to write, make sure that you read carefully the text box at the top of the

essay titles for each section. Don’t treat them as just part of the landscape. Keeping an

eye on the areas and techniques examiners expect to see referred to will remind you

of the need to respect technical vocabulary in the analysis of your chosen text. Revise

this area carefully before the exam, ensuring you know exactly the terms referred to in

each one. Are you, for instance, entirely sure you know the difference between ‘plot’ and

‘theme’? Check.

Now, examiners are human, despite what you may think. They are well aware that

you are writing under pressure and that there will no doubt be the odd slip which you

probably would not make under normal circumstances. You must, however, aim for a

script that is as technically accurate as you can possibly make it. Watch out for trying to

make commas do the job of full stops; you know from your Close Reading how colons

and semi-colons work, so make them work for you here. Simple things like careless use

of the apostrophe can betray a lack of basic literacy, which is not a good way to impress

an examiner. What is hugely important, too, is that examiners can actually read your

writing. They are experienced professionals, but there are limits to what even they can

make out. If an examiner simply cannot decipher an incisive point, you are unlikely to be

rewarded for it.

Dealing with quotations

Be careful how you lay out your quotations. While there are no absolute rules for this,

basic conventions such as paragraphing and indenting longer quotations, integrating

shorter ones into a sentence should be observed.

Longer quotations

Here use a colon, drop a line and lay out your quotation as in the original text.

So you might write:

Duncan’s generosity of nature to all his subjects is suggested when the sergeant conveying

the news of Macbeth’s victories collapses. The king’s response is immediate:

‘So well thy words become thee as thy wounds;

They smack of honour both. Go get him surgeons.’

Short quotations.

If it is a short quotation, say 2/3/4 words, you simply weave it into your own text, using

inverted commas to show these words are not your own.

The king himself refers to him as ‘valiant cousin! worthy gentleman!’ and later as ‘noble

Macbeth’.

What are the pitfalls of using quotations?

All this may sound quite straightforward, but there are conventions attached to quoting

you need to respect. You cannot assume readers know everything that is going on in

your head, so you need to help them.

Pitfall 1

It is no use saying, as students sometimes do:

Duncan has a great generosity of nature:

‘Go get him surgeons.’

then pass on to the next point. In other words, you need to make sure readers

understand the context from which the quotations are taken. This will require short

phrases which lead readers into the quotation in a way that helps them make sense of

the point you are making.

A better use of this quotation might be:

Duncan has a great generosity of nature. Seeing a badly-wounded messenger collapse

in front of him, he sees to it personally that the humble sergeant be taken care of:

‘Go get him surgeons.’

Here the remark has been contextualized to lead readers into the quotation and to

understand better the point you are making.

You will find that some well-chosen short quotations are often more effective in

supporting your statements than longer ‘clunkier’ ones in which the exact point you

are hoping to make may well get lost somewhere among the many lines. If these shorter

quotations can be woven seamlessly into your own text, you will be presenting the

examiner with a sophisticated texture of analysis which will stand you in good stead.

Pitfall 2

Make sure your lead-in to the quotation does not simply repeat the content of the

quotation. For example,

Jimmy tells his wife he is going on a journey but he is not sure where he is going:

‘Ah’m gaun on a journey but ah don’t know where ah’m gaun.

The lead-in here might better be:

Jimmy comes across to his wife-and to readers- as rather vague about his plans for taking

his study of Buddhism forward:

‘Ah’m on a journey but ah don’t know where ah’m gaun.

[‘Buddha Da’ Anne Donovan. Canongate.]

In other words, a thoughtful lead-in does more than simply help the reader to

understand the quotation’s context, it can also underline your ability to evaluate its

significance.

The hazards of the prose section

In an exam of this importance, you need to read all questions very carefully, but with

questions in the Prose section you need to be doubly cautious. Some questions will

refer to novels, some to novels or short stories, some to two short stories, others to a

non-fiction text or travel writing.

Should you get in a muddle over this and write about a short story instead of a novel

or some such similar selection confusion, your script will be referred to the Principal

Assessor who will apply an appropriate penalty. This penalty could be the difference

between your passing and failing the exam, so you should check carefully that the text

you are writing about is entirely suitable for the question.

Page 20: PAST PAPER QUESTIONS FOR MAY EXAMS - Elgin · PDF filePAST PAPER QUESTIONS FOR MAY EXAMS Of cial SQA past paper questions for Maths, Biology, Physics and Chemistry - from Standard

20 | Official SQA Past Papers Supplement - English

Bright Red Official SQA Past Papers Higher English

ISBN: 978-1-84948-213-4£8.99

Writing about poetry

In essays on poetry there are two common faults which you should try to avoid.

Don’t work your way through a poem line-by-line. (Markers and Examiners call this

‘the guided tour’.) While you mustn’t ignore significant sections of the poem or distort

its overall idea, your essay should be shaped to answer the question relevantly and not

dominated by an insistence on examining every line in order.

The second pitfall is allowing your understanding and appreciation of poetic

techniques to take the place of saying what the effect of all these techniques is on the

reader. In other words, spotting onomatopoeia at work will not get you any marks

unless you can say what it is contributing to the reader’s experience and the impact it

is having on the

poem as a whole.

Extended advice on many aspects of the Critical Essay paper can be found on SQA’s website

(see ‘Candidate Guidance Documents’

on the dedicated English page).

Page 21: PAST PAPER QUESTIONS FOR MAY EXAMS - Elgin · PDF filePAST PAPER QUESTIONS FOR MAY EXAMS Of cial SQA past paper questions for Maths, Biology, Physics and Chemistry - from Standard

Official SQA Past Papers Supplement - English | 21

Use SCOT2 at www.brightredpublishing.co.uk for 30% off and free post and packaging!

ADVANCED HIGHER

ENGLISHAbout the author

Iain Valentine is principal teacher of English at Elgin Academy. He has taught English at all levels for 24 years and was the SQA’s Principal Assessor for Standard Grade English from 2007 – 2011.

He is now the Principal Assessor for Advanced Higher English.

The Advanced Higher qualification in English enables you to specialise in various areas of study. You will be given the opportunity to closely study some of the great works of English literature and you will gain considerable ability in think-ing and working independently. Your language skills will be sophisticated, as will be your analytical skills.

There are three component parts to the Advanced Higher English Course: two mandatory Units (English: Specialist Study and English: Literary Study) and one optional Unit (to be selected from English: Language Study, English: Textual Analysis, English: Reading the Media, English: Creative Writing).

Page 22: PAST PAPER QUESTIONS FOR MAY EXAMS - Elgin · PDF filePAST PAPER QUESTIONS FOR MAY EXAMS Of cial SQA past paper questions for Maths, Biology, Physics and Chemistry - from Standard

22 | Official SQA Past Papers Supplement - English

Specialist Study - DissertationThe dissertation is worth 40% of your overall award—so it’s important to take it

very seriously.

Authors, texts and topics that are central to your work in one component of course assessment (for example, Literary Study) may not be used in any other component of course assessment (for example, your Specialist Study Dissertation). You will be required to record your Specialist Study Dissertation texts and topic on your answer booklet in the examination. You have to make an independent study of and produce a dissertation on an aspect or aspects of language or literature or media or some combination of these. Your study should explore a limited area and examine it in detail with lots of appropriate supporting evidence.

It should be noted that texts and topics:

. Must be personally selected by you (under the guidance of your teacher or lecturer). Must be accepted by your centre as suitable choices. Must not be the subject of teaching in this unit. Must not be the subject of teaching or assessment in other units of the Advanced Higher English course or in the units of other courses

. Make clear to you the procedures that must apply in order to meet deadlines and evidence requirements. Record your progress at different stages in the production of the dissertation in order to ensure the authenticity of your work

The dissertation you produce must be between 3500 and 4500 words in length, including quotations but excluding footnotes and bibliography.

Creative Writing FolioThe Creative Writing Folio is worth 30% of your overall award—so it’s important to take it very seriously too. You must show that you can write creatively in more than one genre by submitting for assessment two pieces of creative writing. The genres from which you can choose are:

. reflective essay. prose fiction. poetry. drama Other than poetry, where length should be appropriate to subject and form, each piece of creative writing should be at least 1000 words in length. You are required to indicate on the Creative Writing folio flyleaf the actual number of words used in each piece. You should also note that, although there is no prescribed maximum length, excessively lengthy pieces are usually self-penalising.

The ExaminationThe SQA examination paper is worth either 30% or 60% of your overall award – another thing to take very seriously! A prelim exam will provide your school with evidence for your final estimated grade. At first glance, the SQA Question Paper may seem daunting—around forty pages long, lots of detailed information to absorb on the opening page, a dozen or so pages and over forty questions in the mandatory Literary Study section alone. But you needn’t panic!

Only one or at most two questions have to be answered. If Creative Writing is your chosen Option, you need only answer one question - which must, of course, be selected from the mandatory Literary Study section of the paper. If you have chosen an Option other than Creative Writing, you will be required to answer two questions. Your first will be, as it is for all candidates, from the mandatory Literary Study section. Your second will be from the Language Study section or the Textual Analysis section or the Reading the Media section. Remember that you have one and a half hours in which to tackle each question. Of course, not all of that time should be spent writing. You should take as much time as you need to think about the question and to plan how you intend to respond to it. Then you should begin writing in as much detail as you think relevant and manageable in approximately one hour’s worth of solid writing.

For the mandatory Literary Study section, you should have studied the work of at least two authors (from different genres - Drama, Poetry, Prose). If you are properly prepared, therefore, for the examination, you should have a choice between the questions set on each of your chosen authors.

Remember that, unlike at Higher, the questions will be specific to the author/specified text(s) – see the examples from the 2009 exam below.

6. LochheadMake a detailed study of the role of La Corbie in Mary Queen of Scots Got Her Head Chopped Off and of Renfield in Dracula.

14. DonneDiscuss the uses Donne makes of aspects of Renaissance learning and discovery in The Good Morrow, The Sun Rising, Aire and Angels and A Valediction: forbidding mourning.

26. Fitzgerald“The world of a Fitzgerald novel is glamorous but essentially shallow; its characters live in an emotional and spiritual vacuum.”In the light of this statement, discuss some of the principal means by which Fitzgerald presents the worlds of The Beautiful and Damned and Tender is the Night.

There is similar choice in the Optional Sections of the paper: Language Study (for which two topics will have been studied), Textual Analysis (for which two genres will have been practised) and Reading the Media (for which two media categories will have been examined). After the Creative Writing folio, Textual Analysis is the most popular option for candidates.

If you have chosen this option, you must remember that this is essentially a reading exercise. It should build on the skills you have already acquired at Higher and it is vital that you offer analysis of the techniques used by the writer and not simply paraphrase what is in the text. Working through past papers will be invaluable preparation for this part of the paper but do not hesitate to ask your teacher for further guidance. See below for examples of questions from the Optional Sections of the paper:

2010 - Section 2, Language StudyTopic C—Multilingualism in contemporary Scotland5. In your own reading and research, what evidence have you found of codeswitching between languages by speakers in contemporary Scotland? In your answer you should consider some of the forms, contexts and purposes of such codeswitching.

2009 - Section 3, Textual Analysis Read carefully the poem The world is too much with us . . . (1807) by William Wordsworth and then answer the question that follows it.

The world is too much with us; late and soon,Getting and spending, we lay waste our powers;Little we see in Nature that is ours;We have given our hearts away, a sordid boon!This Sea that bares her bosom to the moon;The winds that will be howling at all hours,And are up-gathered now like sleeping flowers;For this, for everything, we are out of tune;It moves us not.—Great God! I’d rather beA Pagan suckled in a creed outworn;So might I, standing on this pleasant lea,Have glimpses that would make me less forlorn;Have sight of Proteus(1) rising from the sea;Or hear old Triton(2) blow his wreathèd horn.

(1) An ancient Greek sea god capable of taking many shapes.(2) An ancient Greek sea god often depicted as trumpeting on a shell.

QuestionWrite a detailed critical analysis of this poem in which you make clear what you consider to be the significant features of its language and form.

Page 23: PAST PAPER QUESTIONS FOR MAY EXAMS - Elgin · PDF filePAST PAPER QUESTIONS FOR MAY EXAMS Of cial SQA past paper questions for Maths, Biology, Physics and Chemistry - from Standard

Official SQA Past Papers Supplement - English | 23

Use SCOT2 at www.brightredpublishing.co.uk for 30% off and free post and packaging!

2011 - Section 4, Reading the MediaCategory A—Film1. “A film should have a beginning, a middle and an end—but not necessarily in that order.”(Jean-Luc Godard)Discuss with reference to the narrative structure of one or more than one film you have studied.

Because you don’t have access to your texts during the examination, you will not be

penalised for minor inaccuracies as you attempt to support your answer with relevant

quotations and references. You should, however, be so familiar with the texts you have

studied that quoting from them is instinctive and natural to you. Learning key quota-

tions by heart is one way of ensuring that you have developed a firm grasp of the cen-

tral concerns and features of the text.

You should write directly in response to the terms of the question you are attempting.

One of the main things examiners are looking for is evidence of your willingness to get

involved, your ability to confront the terms of the question - to meet its implications

head on and to deploy your knowledge of the text to provide convincing supporting

evidence for the line of thought you are seeking to develop.

Bright Red Official SQA Past PapersAdvanced Higher English

ISBN: 978-1-84948-230-1£8.99

Can’twait

SQAMywww.mysqa.infoRegister now to get your exam results first thing on 7th August by text or email, at home or on holiday.

SAM is here to help on Facebook, the Student Room or Twitter.

Customer Contact Centre T: 0845 279 1000 E: [email protected]

Don’twait

DON’T SWEAT IT

www.mysqa.info SQAMy

Don’t Wait – register now to receive your exam results by text or e-mail.

www.mysqa.info SQAMy

Get your

results

to get your results on the 4th of August

by text or e-mail, at home or on holiday. So you can relax

when your paper certificate arrives on the 5th of August.

www.mysqa.info SQAMy

Get your

results

to get your results on the 4th of August

by text or e-mail, at home or on holiday. So you can relax

when your paper certificate arrives on the 5th of August.

Page 24: PAST PAPER QUESTIONS FOR MAY EXAMS - Elgin · PDF filePAST PAPER QUESTIONS FOR MAY EXAMS Of cial SQA past paper questions for Maths, Biology, Physics and Chemistry - from Standard

24 | Official SQA Past Papers Supplement - English

STANDARD GRADE INTERMEDIATE HIGHER ADV HIGHER

ENGLISH